20.) The area of a circle is given by the equation A = nr2. If the radius of a circle is equal to 6 centimeters,
which of the following is closes to the area of the circle? (Use it = 3.14.)

113.04
18.84
36
28.26

please provide explanation​

Answers

Answer 1

Answer:

113.04 cm^2

Step-by-step explanation:

The area of a circle is

A = pi r^2

We know the radius is 6 cm

A = 3.14 * 6^2

A = 3.14 * 36

A =113.04


Related Questions

HELP ME PLEASE!!!!!!!!! WORTH 100 POINTS WILL FOLLOW AND RATE BRAINLIEST ANSWER!!!!!!!!!!!! If the function f(x) has a domain of (a,b] and a range of [c,d), then what is the domain and range of g(x)=m×f(x)+n?(1 point) A.The domain of g(x) is (ma+n,mb+n], and the range is [c,d). B.The domain of g(x) is (a,b], and the range is [mc+n,md+n). C.The domain of g(x) is (a,b], and the range is [c,d). D.The domain of g(x) is (ma+n,mb+n], and the range is [mc+n,md+n).

Answers

Greetings from Brasil...

If  domain is (a,b] and the range is [c,d), so:

f(a) = c

f(b) = d

so

g(x) = m × f(x) + n

g(a) = m × f(a) + n       how f(a) = c, then

g(a) = mc + n

g(b) = m × f(b) + n

g(b) = md + n

So

Domain = (a; b]Range = [mc + n; md + n)

Sherina wrote and solved the equation. x minus 56 = 230. x minus 56 minus 56 = 230 minus 56. x = 174. What was Sherina’s error?

Answers

Answer:

  subtracting 56 instead of adding (or adding wrong)

Step-by-step explanation:

She wrote ...

  x - 56 = 230

  x - 56 - 56 = 230 -56 . . . . correct application of the addition property*

  x = 230 -56 . . . . . . . . . . . . incorrect simplification

Correctly done, the third line would be ...

  x -112 = 174

This would have made Sherina realize that the error was in subtracting 56 instead of adding it. The correct solution would be ...

  x - 56 + 56 = 230 + 56 . . . using the addition property of equality

  x = 286 . . . . . . . . . . . . . . . . correct simplification on both sides

__

There were two errors:

  1) incorrect strategy --- subtracting 56 instead of adding

  2) incorrect simplification --- simplifying -56 -56 to zero instead of -112

We don't know whether you want to count the error in thinking as the first error, or the error in execution where the mechanics of addition were incorrectly done.

_____

* The addition property of equality requires the same number be added to both sides of the equation. Sherina did that correctly. However, the number chosen to be added was the opposite of the number that would usefully work toward a solution.

Answer:

D: Sherina should have added 56 to both sides of the equation.

Step-by-step explanation:

I got a 100% on my test.

I hope this helps.

Which formula used in probability to find Independence question

Answers

Answer:

Events A and B are independent if the equation P(A∩B) = P(A) · P(B) holds true. You can use the equation to check if events are independent; multiply the probabilities of the two events together to see if they equal the probability of them both happening together.

Answer:

Events are independent if the outcome of one effect does not effect the outcome

Step-by-step explanation:

1
1
A baseball weighs approximately
3
pound. A golf ball weighs about pound.
10
What expression can be used to find the combined weight of a baseball and a golf ball?​

Answers

Answer:

6 pounds 10 ounces

Step-by-step explanation:

I take this to mean "a baseball weighs approximately 3 pounds and a golf ball three pounds ten ounces."

Adding these two weights together, we get 6 pounds 10 ounces.

PLEASE HELP 30 POINTS

How long will it take in hours for a car traveling from Tucson to Phoenix (120 km)
to reach Phoenix at a rate of 10km/hr.? How long would it take that car to circle the Earth
at the equator? (c= 2 nr) rof earth is 6,378 km.

Answers

Answer:

1. It would take the car to get from Tucson to Phoenix 12 hours.

2. for the car to go around the equator it would take 637 hours if it is still travelling at 10km/hr.

hope this helps

Step-by-step explanation:

1. 120 km divided by 10 = 12 hours

Find the Vertical asymptotes of the graph of f
[tex]f(x) = \frac{x + 2}{ {x}^{2} - 4}[/tex]

Answers

Answer:

x = 2 and x = -2

Step-by-step explanation:

To find the vertical asymptotes, set the denominator equal to zero and solve for x:

vertical asymptotes are x = 2 and x = -2

List the sides in order from the largest to the smallest. A. XY, YW, WX B. XY, WX, YW C. WX, YW, XY D. WX, XY, YW

Answers

Answer:

Option (D)

Step-by-step explanation:

By applying Sine rule in the given triangle WXY,

[tex]\frac{\text{SinW}}{\text{XY}}=\frac{\text{SinY}}{\text{XW}}=\frac{\text{SinX}}{\text{WY}}[/tex]

[tex]\frac{\text{Sin59}}{\text{XY}}=\frac{\text{Sin82}}{\text{XW}}=\frac{\text{Sin39}}{\text{WY}}[/tex]

[tex]\frac{\text{Sin59}}{\text{XY}}=\frac{\text{Sin82}}{\text{XW}}[/tex]

[tex]\frac{\text{XW}}{\text{XY}}=\frac{\text{Sin82}}{\text{Sin59}}[/tex]

      = 1.1489

XW : XY ≈ 1.15 : 1

[tex]\frac{\text{Sin59}}{\text{XY}}=\frac{\text{Sin39}}{\text{WY}}[/tex]

[tex]\frac{\text{XY}}{\text{WY}}=\frac{\text{Sin59}}{\text{Sin39}}[/tex]

[tex]\frac{\text{XY}}{\text{WY}}=\frac{1.36}{1}[/tex]

[tex]\frac{\text{XY}}{\text{WY}}=\frac{\frac{1}{1}}{\frac{1}{1.36} }[/tex]

[tex]\frac{\text{XY}}{\text{WY}}=\frac{1}{0.7342}[/tex]

XY : WY = 1 : 0.7342

XW : XY : WY = 1.15 : 1 : 0.7342

Therefore, WX > XY > WY

Option (D). will be the correct option.

If x = -1 then how much is 2x - 1

a) 1
b) -3
c) -2​
hurry please need to turn in 10 min

Answers

Answer: -3

Step-by-step explanation: 2x = -2 then you subtract 1 from that which is the same as adding negative one so -2 - 1 or -2 + -1 = -3

The Freeman family is barbecuing veggie burgers, corn cobs, and mushroom caps in their local park. If 3 8 of the items barbecued are veggie burgers, and 1 3 of the items barbecued are corn cobs, what fraction of barbecued items are mushroom caps?

Answers

Answer:

The answer is below

Step-by-step explanation:

The Freeman family barbecued veggie burgers, corn cobs, and mushroom caps. 3/8 of the items barbecued are veggie burgers, and 1/3 of the items barbecued are corn cobs.

Let the total number of berbecued items be x. Therefore:

x = barbecued veggie burgers + barbecued corn cobs + barbecued mushroom caps

Barbecued veggie burgers = (3/8)x, barbecued corn cobs = (1/3)x, Let barbecued mushroom caps be y

Substituting:

x = (3/8)x + (1/3)x + y

Multiply through by 24

24x = 9x + 8x + 24y

24x = 17x + 24y

24y = 24x - 17x

24y = 7x

y = (7/24)x

barbecued mushroom caps = (7/24) of items

7/24 of the items barbecued are mushroom caps

Using fractions, it is found that the fraction of barbecued items that are mushroom caps is of [tex]\frac{7}{24}[/tex].

---------------------------

The total proportion of all products is 100% = 1.The fraction corresponding to veggie burgers is [tex]\frac{3}{8}[/tex].The fraction corresponding to corn cobs is [tex]\frac{1}{3}[/tex].The fraction corresponding to mushroom caps is x.

---------------------------

Thus:

[tex]\frac{3}{8} + \frac{1}{3} + x = 1[/tex]

Solving for x, we find the fraction of mushroom caps.The least common multiple of 3 and 8 is 24.

Then:

[tex]\frac{3\times3 + 8\times1 + 24x}{24} = 1[/tex]

[tex]\frac{17 + 24x}{24} = 1[/tex]

[tex]17 + 24x = 24[/tex]

[tex]24x = 7[/tex]

[tex]x = \frac{7}{24}[/tex]

The fraction of barbecued items that are mushroom caps is of [tex]\frac{7}{24}[/tex].

A similar problem is given at https://brainly.com/question/4231000

PLEASE HELP !!! (5/5) -50 POINTS-

Answers

Answer:

at least one solution

Step-by-step explanation:

Consistent solutions have at least one solution, but may have more than one solution.  Intersecting lines and  Lines that are the same are consistent solutions

Answer:

[tex]\boxed{Atleast\ one \ Solution}[/tex]

Step-by-step explanation:

A consistent system of equations have at least one solution. It can be more than that. There are no compulsions.

What is the difference? Complete the equation. -1 2/5 - (-4/5) = ?

Answers

Answer:

First convert them which will be

-7/5 - (-4/5)

so when you subtract a negative number from negative number they actually subtract ex = -4-(-2) = -2

so its simply 7/5-4/5 then add a negative sign

so

3/5

now add negative sign so

-3/5

(12x^(2)+x-35)-:(4x+17)

Answers

Answer:

(3x-5)(4x+7) / 4x + 17

Step-by-step explanation:

Rewrite the division as a fraction

12 x ^2 + x-35 / 4x+17

Factor by grouping

(3x-5)(4x+7) / 4x + 17

Hope this was the answer you were looking for

write a letter to your friend in Ghana stating your experience in your presentation school in nigeria​

Answers

Answer:

hi Ghana how are you doing I am fine here. I really miss u and my friends in the old.U know what in Nigeria this school is really awesome and fantastic we have a swimming pool here and we can go to trip and we can have many things here I really loved this school.

at starting I was not have any friends and know I have many friends. But I really miss u this is what about our . Come to my house I can show you my school it is very near to my house .

Ur friend

writ ur name

Marco is investigating some of the business models of SureSpin, one of Faster Fidget's top competitors.

He has learned that they model their cost of production for one type of spinner with the function C(x) =13,450 + 1.28x, where x is the number of spinners produced. Interpret the model to complete the

statement.

Type the correct answer in each box. Use numerals instead of words. Based on the model, the fixed cost of production is $?

Answers

Answer:

$13,450

Step-by-step explanation:

The fixed cost of production is $13,450, this is because a fixed cost of production is the amount of cost that does not change with an increase or decrease in the amount of the goods or services produced. Fixed cost of production are paid by companies. It is one of the two component of the total cost of goods or services along with the variable cost.

In regard to the information given in the  question, no matter how many spinners the company produces, the fixed cost will remain the same.

Assuming x is the variable cost which signifies the number of spinners produced, this literally implies that the cost to produce each spinner is $1.28 and the fixed cost which is independent  of the production is $13,450.

Hence, the  fixed cost of production is $13,450.

What is the rate of change from x = 0 to x = pi over 2 ? (6 points) trig graph with points at (0, -4) and (pi over 2, 0) and (pi, 4) and (3 pi over 2, 0) and (2 pi, -4)

Answers

Answer: [tex]\dfrac{8}{\pi}[/tex] .

Step-by-step explanation:

We know that the rate of change of function f(x) from x=a to x= b is given by :-

[tex]k=\dfrac{f(b)-f(a)}{b-a}[/tex]

The given points on graph  :  (0, -4) and (pi over 2, 0) and (pi, 4) and (3 pi over 2, 0) and (2 pi, -4).

The rate of change from x = 0 to x = pi over 2 will be :-

[tex]\dfrac{0-(-4)}{\dfrac{\pi}{2}-0}=\dfrac{4}{\dfrac{\pi}{2}}[/tex]     [By using points (0, -4) and (pi over 2, 0) ]

[tex]=\dfrac{8}{\pi}[/tex]

Hence, the rate of change from x = 0 to x = pi over 2 is [tex]\dfrac{8}{\pi}[/tex] .

pls what is the nearest 100 of 49​

Answers

Answer:

the nearest hundred is 50

In a triangle, the sum of two angles equals the third, Find the measure of the third angle.
A.45 degree
B.60 degree
C.90 degree
D.30 degree

Answers

Answer:

C.90 degree

Step-by-step explanation:

45 + 45 + 90 = 180

90 = 45 + 45

-x + 3y = 3

x - 3y = 3

Does this system have a solution?

Answers

Answer:

No solution

Step-by-step explanation:

Slope-Intercept Form: y = mx + b

Step 1: Write out systems of equations

-x + 3y = 3

x - 3y = 3

Step 2: Rewrite equations into slope-intercept form

3y = 3 + x

y = 1 + x/3

-3y = 3 - x

y = -1 + x/3

Step 3: Rewrite systems of equations

y = x/3 + 1

y = x/3 - 1

Since we have the same slope for both equations but different y-intercepts, we know that both lines are parallel. If that is the case, they will never touch or intersect each other. Therefore, we have no solution.

100 students are interviewed to see which of biology, chemistry or physics they prefer.
59 of the students are girls. 35 of the girls like biology best.
2 of the boys prefer physics.
6 out of the 30 who prefer chemistry are girls.
What percentage of the students prefer biology?

Answers

Answer:

50%

Step-by-step explanation:

Girls                                            Boys

total:                 59                     total:                              41

- Chemistry       35                   - Physics                           2

                       = 24                                    =                     39

                                                 - Chemistry ( 30 - 6 )      24

                                                                   =                     15

Total boys and girls for Biology = 35 + 15 = 50

% = 50/100*100

   = 50%

Hope it helps and also mark it as brainliest!!!!

What is the result of question?

Answers

Answer: B. 26x+270 less than or equal to 1,325

Explanation:
26, people attending
x, money spent in lunch for each guest
270, cost of renting the meeting room
Less than or equal to, because 1,325 is the max budget
1,325, is the budget


Hope this helps <3

Answer:

B

Step-by-step explanation:

x can not be greater than (1,325-270)/26 because $270 is fixed for the rental

(Algebra)
Plz help me ASAP!! I’ll be so grateful!

Answers

Answer:

y > 1

Step-by-step explanation:

-2(7 + y) > -8(y + 1)

-14 -2y > -8y -8

-2y +8y > -8 +14

6y > 6

6y/6 > 6/6

y > 1

what should be added to 66.778 get 78.2​

Answers

Answer:

11.422

Step-by-step explanation:

[tex]78.2 - 66.778 \\ = 11.422[/tex]

7 less than the quotient of a number and 3 is 5. Find the number.

Answers

Answer:

The answer is 36

Step-by-step explanation:

Let the number be x

7 less than the quotient of a number and 3 is written as

[tex] \frac{x}{3} - 7[/tex]

The result is 5

So we have

[tex] \frac{x}{3} - 7 = 5[/tex]

Move - 7 to the right side of the equation

That's

[tex] \frac{x}{3} = 7 + 5[/tex][tex] \frac{x}{3} = 12[/tex]

Multiply both sides by 3 to make x stand alone

We have

[tex]3 \times \frac{x}{3} = 12 \times 3[/tex]

We have the final answer as

x = 36

Hope this helps you

A study of 200 computer service firms revealed these incomes after taxes: Income After Taxes Number of Firms Under $1 million 102 $1 million up to $20 million 61 $20 million or more 37 What is the probability that a particular firm selected has $1 million or more in income after taxes

Answers

Answer:

The probability that a particular firm selected has $1 million or more in income after taxes is 49%.

Step-by-step explanation:

We are given a study of 200 computer service firms revealed these incomes after taxes below;

         Income After Taxes                  Number of Firms

           Under $1 million                              102

      $1 million up to $20 million                    61

           $20 million or more                          37      

                 Total                                           200    

Now, the probability that a particular firm selected has $1 million or more in income after taxes is given by;

Total number of firms = 102 + 61 + 37 = 200

Number of firms having $1 million or more in income after taxes = 61 + 37 = 98  {here under $1 million data is not include}

So, the required probability =  [tex]\frac{\text{Firms with \$1 million or more in income after taxes}}{\text{Total number of firms}}[/tex]

                                           =  [tex]\frac{98}{200}[/tex]

                                           =  0.49 or 49%

The probability that a particular firm selected has $1 million or more in income after taxes is 0.49 or 49%.

What is probability?

Probability means possibility. It deals with the occurrence of a random event. The value of probability can only be from 0 to 1. Its basic meaning is something is likely to happen. It is the ratio of the favorable event to the total number of events.

A study of 200 computer service firms revealed these incomes after taxes:

Income After Taxes Number of Firms Under

$1 million 102

$1 million up to $20 million 61

$20 million or more 37.

Then the total event will be

Total event = 102 + 37 +61 = 200

The probability that a particular firm selected has $1 million or more in income after taxes will be

Favorable event = 37 + 61 = 98

Then the probability will be

[tex]\rm P = \dfrac{98}{200} \\\\P = 0.49 \ or \ 49 \%[/tex]

More about the probability link is given below.

https://brainly.com/question/795909

Find the length of AB¯¯¯¯¯¯¯¯ A. 19.56 B. 51.86 C. 42.99 D. 34.98

Answers

Answer:

Apllying cos on the triangle

cos(angle)= Base/ Hyp

cos(34)= 29/ AB

AB= 29/0.8290

AB=34.98

Step-by-step explanation:

The length of AB is 34.98 units which the correct answer would be an option (D).

What is the right triangle?

A right triangle is defined as a triangle in which one angle is a right angle or two sides are perpendicular.

What are Trigonometric functions?

Trigonometric functions are defined as the functions which show the relationship between the angle and sides of a right-angled triangle.

Given that ΔABC

∠C = 90°

Here base = BC = 29 units and hypotenuse = AB

To determine the length of AB

Apply the cosine on the given right triangle

⇒ cos(θ) = Base/hypotenuse

⇒ cos(34) = 29/ AB

∴ cos(34°) = 0.8290

⇒ 0.8290 = 29/ AB

⇒ AB= 29/0.8290

⇒ AB = 34.98 units

Hence, the length of AB is  34.98 units

Learn more about Trigonometric functions here:

https://brainly.com/question/6904750

#SPJ2

A box contain 12 balls in which 4 are white 3 are blue and 5 are red.3 balls are drawn at random from the box.find the chance that all three are selected​

Answers

Answer:

3/11

Step-by-step explanation:

From the above question, we have the following information

Total number of balls = 12

Number of white balls = 4

Number of blue balls = 3

Number of red balls = 5

We solve this question using combination formula

C(n, r) = nCr = n!/r!(n - r)!

We are told that 3 balls are drawn out at random.

The chance/probability of drawing out 3 balls = 12C3 = 12!/3! × (12 - 3)! = 12!/3! × 9!

= 12 × 11 × 10 × 9 × 8 × 7 × 6 × 5 × 4 × 3 × 2 × 1/(3 × 2 × 1) × (9 × 8 × 7 × 6 × 5 × 4 × 3 × 2 × 1)

= 220 ways

The chance of selecting 3 balls at random = 220

To find the chance that all the three balls are selected,

= [Chance of selecting (white ball) × Chance of selecting(blue ball) × Chance of selecting(red balls)]/ The chance/probability of drawing out 3 balls

Chance of selecting (white ball)= 4C1

Chance of selecting(blue ball) = 3C1 Chance of selecting(red balls) = 5C1

Hence,

= [4C1 × 3C1 × 5C1]/ 220

= 60/220

= 6/22

= 3/11

The chance that all three are selected is = 3/11

The ages of some lectures are 42,54,50,54,50,42,46,46,48 and 48.Calculate the:
(a)Mean Age.
(b)Standard deviation.

Answers

Answer:

The mean age is 48

The standard deviation is 4

Step-by-step explanation:

The answer is, (a) mean age is 48.

                          (b)  standard deviation is 4.

What is a mean age?Average age of the population calculated as the arithmetic mean.Another parameter determining the average age of the population is the median age.

What does standard deviation of age mean?In general, the standard deviation tells us how far from the average the rest of the numbers tend to be, and it will have the same units as the numbers themselves. If, for example, the group {0, 6, 8, 14} is the ages of a group of four brothers in years, the average is 7 years and the standard deviation is 5 years.

How do you find the mean age?To find the mean add all the ages together and divide by the total number of children.

Learn more about mean age and standard deviation here:

https://brainly.com/question/475676

#SPJ2

The amount of money spent on textbooks per year for students is approximately normal.
A. To estimate the population mean, 19 students are randomly selected the sample mean was $390 and the standard deviation was $120. Find a 95% confidence for the population meam.
B. If the confidence level in part a changed from 95% 1 to 1999%, would the margin of error for the confidence interval:
1. decrease.
2. stay the same.
3. increase not.
C. If the sample size in part a changed from 19% 10 to 22, would the margin of errot for the confidence interval:
1. decrease.
2. stay the same.
3. increase
D. To estimate the proportion of students who purchase their textbookslused, 500 students were sampled. 210 of these students purchased used textbooks. Find a 99% confidence interval for the proportion of students who purchase used text books.

Answers

Answer:

(A) A 95% confidence for the population mean is [$332.16, $447.84] .

(B) If the confidence level in part (a) changed from 95% to 99%, then the margin of error for the confidence interval would increase.

(C) If the sample size in part (a) changed from 19 to 22, then the margin of error for the confidence interval would decrease.

(D) A 99% confidence interval for the proportion of students who purchase used textbooks is [0.363, 0.477]  .

Step-by-step explanation:

We are given that 19 students are randomly selected the sample mean was $390 and the standard deviation was $120.

Firstly, the pivotal quantity for finding the confidence interval for the population mean is given by;

                             P.Q.  =  [tex]\frac{\bar X-\mu}{\frac{s}{\sqrt{n} } }[/tex]  ~ [tex]t_n_-_1[/tex]

where, [tex]\bar X[/tex] = sample mean = $390

            s = sample standard deviation = $120

            n = sample of students = 19

            [tex]\mu[/tex] = population mean

Here for constructing a 95% confidence interval we have used a One-sample t-test statistics because we don't know about population standard deviation.

So, 95% confidence interval for the population mean, [tex]\mu[/tex] is ;

P(-2.101 < [tex]t_1_8[/tex] < 2.101) = 0.95  {As the critical value of t at 18 degrees of

                                               freedom are -2.101 & 2.101 with P = 2.5%}  

P(-2.101 < [tex]\frac{\bar X-\mu}{\frac{s}{\sqrt{n} } }[/tex] < 2.101) = 0.95

P( [tex]-2.101 \times {\frac{s}{\sqrt{n} } }[/tex] < [tex]{\bar X-\mu}[/tex] < [tex]2.101 \times {\frac{s}{\sqrt{n} } }[/tex] ) = 0.95

P( [tex]\bar X-2.101 \times {\frac{s}{\sqrt{n} } }[/tex] < [tex]\mu[/tex] < [tex]\bar X+2.101 \times {\frac{s}{\sqrt{n} } }[/tex] ) = 0.95

95% confidence interval for [tex]\mu[/tex] = [ [tex]\bar X-2.101 \times {\frac{s}{\sqrt{n} } }[/tex] , [tex]\bar X+2.101 \times {\frac{s}{\sqrt{n} } }[/tex] ]

                        = [ [tex]\$390-2.101 \times {\frac{\$120}{\sqrt{19} } }[/tex] , [tex]\$390+2.101 \times {\frac{\$120}{\sqrt{19} } }[/tex] ]

                        = [$332.16, $447.84]

(A)  Therefore, a 95% confidence for the population mean is [$332.16, $447.84] .

(B) If the confidence level in part (a) changed from 95% to 99%, then the margin of error for the confidence interval which is [tex]Z_(_\frac{\alpha}{2}_) \times \frac{s}{\sqrt{n} }[/tex] would increase because of an increase in the z value.

(C) If the sample size in part (a) changed from 19 to 22, then the margin of error for the confidence interval which is [tex]Z_(_\frac{\alpha}{2}_) \times \frac{s}{\sqrt{n} }[/tex]  would decrease because as denominator increases; the whole fraction decreases.

(D) We are given that to estimate the proportion of students who purchase their textbooks used, 500 students were sampled. 210 of these students purchased used textbooks.

Firstly, the pivotal quantity for finding the confidence interval for the population proportion is given by;

                             P.Q.  =  [tex]\frac{\hat p-p}{\sqrt{\frac{\hat p(1-\hat p)}{n} } }[/tex]  ~ N(0,1)

where, [tex]\hat p[/tex] = sample proportion students who purchase their used textbooks = [tex]\frac{210}{500}[/tex] = 0.42    

            n = sample of students = 500

            p = population proportion

Here for constructing a 99% confidence interval we have used a One-sample z-test statistics for proportions

So, 99% confidence interval for the population proportion, p is ;

P(-2.58 < N(0,1) < 2.58) = 0.99  {As the critical value of z at 0.5%

                                               level of significance are -2.58 & 2.58}  

P(-2.58 < [tex]\frac{\hat p-p}{\sqrt{\frac{\hat p(1-\hat p)}{n} } }[/tex] < 2.58) = 0.99

P( [tex]-2.58 \times {\sqrt{\frac{\hat p(1-\hat p)}{n} } }[/tex] < [tex]{\hat p-p}[/tex] < [tex]2.58 \times {\sqrt{\frac{\hat p(1-\hat p)}{n} } }[/tex] ) = 0.99

P( [tex]\hat p-2.58 \times {\sqrt{\frac{\hat p(1-\hat p)}{n} } }[/tex] < p < [tex]\hat p+2.58 \times {\sqrt{\frac{\hat p(1-\hat p)}{n} } }[/tex] ) = 0.99

99% confidence interval for p = [ [tex]\hat p-2.58 \times {\sqrt{\frac{\hat p(1-\hat p)}{n} } }[/tex] , [tex]\hat p+2.58 \times {\sqrt{\frac{\hat p(1-\hat p)}{n} } }[/tex] ]

= [ [tex]0.42 -2.58 \times {\sqrt{\frac{0.42(1-0.42)}{500} } }[/tex] , [tex]0.42 +2.58 \times {\sqrt{\frac{0.42(1-0.42)}{500} } }[/tex] ]

= [0.363, 0.477]

Therefore, a 99% confidence interval for the proportion of students who purchase used textbooks is [0.363, 0.477]  .

The sum of two polynomials is 10a^2b^2-8a^2b+6ab^2-4ab+2 if one addend is -5a^2b^2+12a^2b-5 what is the other addend

Answers

Answer:

The other addend is [tex]15\cdot a^{2}\cdot b^{2}-20\cdot a^{2}\cdot b + 6\cdot a \cdot b^{2}-4\cdot a \cdot b +7[/tex].

Step-by-step explanation:

The other addend is determined by subtracting [tex]-5\cdot a^{2}\cdot b^{2}+12\cdot a^{2}\cdot b-5[/tex] from [tex]10\cdot a^{2}\cdot b^{2}-8\cdot a^{2}\cdot b + 6\cdot a\cdot b^{2}-4\cdot a \cdot b + 2[/tex]:

[tex]x = 10\cdot a^{2}\cdot b^{2}-8\cdot a^{2}\cdot b + 6\cdot a \cdot b^{2}-4\cdot a \cdot b + 2 - (-5\cdot a^{2}\cdot b^{2}+12\cdot a^{2}\cdot b -5)[/tex]

[tex]x = 10\cdot a^{2}\cdot b^{2}-8\cdot a^{2}\cdot b + 6\cdot a \cdot b^{2}-4\cdot a \cdot b +2 +5\cdot a^{2}\cdot b^{2}-12\cdot a^{2}\cdot b+5[/tex]

[tex]x = (10\cdot a^{2}\cdot b^{2}+5\cdot a^{2}\cdot b^{2})-(8\cdot a^{2}\cdot b+12\cdot a^{2}\cdot b)+6\cdot a \cdot b^{2}-4\cdot a \cdot b +7[/tex]

[tex]x = 15\cdot a^{2}\cdot b^{2}-20\cdot a^{2}\cdot b + 6\cdot a \cdot b^{2}-4\cdot a \cdot b +7[/tex]

The other addend is [tex]15\cdot a^{2}\cdot b^{2}-20\cdot a^{2}\cdot b + 6\cdot a \cdot b^{2}-4\cdot a \cdot b +7[/tex].

Answer:

A

Step-by-step explanation:

Ax + By = C for x. plz sove

Answers

Answer:

[tex]\boxed{\boxed{ x=\frac{C-By}{A}; A\neq 0}}[/tex]

Step-by-step explanation:

[tex]Ax+By=C\\\\Ax+By-By=C-By\\\\Ax=C-By\\\\\frac{Ax=C-By}{A}\\\\\boxed{ x=\frac{C-By}{A}; A\neq 0}[/tex]

Hope this helps.

Other Questions
88 feet/second = 60 miles/hour. How many feet per second is 1 mile? (Hint: divide both side of the equation by the same amount.) Why did USA join the First World War? Winnwbagel corp. currently sells 25,200 motor homes per year at 37,800 each, and 10,080 luxury motor coaches per year at $71,400 each. The company wants to introduce a new portable camper to fill out its product line., it hopes to sell 15,960 of these campers per year at $10,080 each. An independent consultant has determined that if the company introduces the new campers, it should boost the sales of its existing motor homes by 3,780 units per year, and reduce the sales of its motor coaches by 756 units per year. What is the amount to use as the annual sales figure when evaluating this project? a. $237,293,280.b. $262,271,520. c. $357,739,200.d. $95739200.e. $160,876,800.f. $249,782,400. United States v. Susan B. Anthony: Justice Ward Hunt's Court Ruling: Which of these inferences about Anthonys defense is best supported by the text? Question 21 options: a) The center of their case was over whether voting is a privilege or a right. b) Anthony attempted to use the constitutional amendment giving African-American men the right to vote to justify her actions, but the judge did not agree. c) The defense believed that voting laws were not well-explained in the Constitution. d) Anthony believed that women being allowed to become lawyers in Illinois meant voting was now legal for everyone in all states. This force governs atomic decay. A population of values has a normal distribution with = 106.9 and =14.5 You intend to draw a random sample of size n=20 What is the probability that a single randomly selected value is less than 109.8? P(X < 109.8) How do you the probability that a sample of size n= 20 is randomly selected with a mean less than 109.8? P(M < 109.8) Also, I have to round the answer to the 4th decimal place. How do I do that? I dont really understand how to solve this A house m by m is surrounded by a walkway m wide. 27 9 1.8 a) Find the area of the region covered by the house and the walkway. b) Find the area of the walkway. Solve |2x+3/4 |=5 1/2 Please help!!!! Select the correct answer. Which of these means of transport is the most economic? A. auto B. metro C. bus D. bicicleta If 2/3 inch on a map corresponds to an actual distance of 9 miles, what distance on the map will represent 21 miles? What were the main points of the Allied "Europe first strategy? Check all that apply. securing international waters through island hopping finalizing development of an atomic bomb liberating Jews from death camps defeating Germany, then focusing on Japan deploying most Allied troops to Europe Last year Harrington Inc. had sales of $325,000 and a net income of $19,000, and its year-end assets were $250,000. The firm's total-debt-to-total-capital ratio was 15.0%. The firm finances using only debt and common equity and its total assets equal total invested capital. Based on the DuPont equation, what was the ROE Choose the group of words in which all are spelled correctly. A. aclaimed, exquisite, fundamentaly B. accommodate, discipline, scavenge C. aparition, expedite, ocassional, D. accesory, dominence, problematic Determine which set of properties correctly describes copper (Cu)?A. Giant structure, conducts electricity, high melting point, soluble in water, malleableB. Malleable, brittle, soluble in oil or gasoline, high melting point, simple structureC. Ionic lattice, conducts electricity, soluble in oil or gasoline, low melting point, ductileD. Malleable, conducts electricity, high melting point, giant structure, metallic lattice The base of a right triangle is increasing at a rate of 2 meters per hour and the height is decreasing at a rate of 3 meters per hour. When the base is 9 meters and the height is 22 meters, then how fast is the HYPOTENUSE changing Evaluate the integral using integration by parts with the indicated choices of u and dv. (Use C for the constant of integration.) 4x2 lnx dx ; u= lnx , dv=4x 2dx A premature infant develops jaundice. Laboratory test results are negative for hemolytic disease of the newborn, but the infants bilirubin level continues to rise. Abnormal urinalysis test results include a dark yellow color, positive Ictotest, and needle-shaped crystals seen on microscopic examination. What is the most probable cause of the infants jaundice? 14. You bought shoes for $60. They were on sale for 40% off their original price.a. Write a proportion to solve for the original price of the shoes.b. Solve the proportion in part (a). According to a survey, typical American spends 154.8 minutes per day watching TV. A survey of 50 Internet users results in a mean time watching TV per day of 128.7 minutes, with a standard deviation of 46.5 minutes. Which appropriate test we should use to determine if Internet users spend less time watching TV